-
Notifications
You must be signed in to change notification settings - Fork 2
/
questionlibd.tex
347 lines (297 loc) · 11.4 KB
/
questionlibd.tex
1
2
3
4
5
6
7
8
9
10
11
12
13
14
15
16
17
18
19
20
21
22
23
24
25
26
27
28
29
30
31
32
33
34
35
36
37
38
39
40
41
42
43
44
45
46
47
48
49
50
51
52
53
54
55
56
57
58
59
60
61
62
63
64
65
66
67
68
69
70
71
72
73
74
75
76
77
78
79
80
81
82
83
84
85
86
87
88
89
90
91
92
93
94
95
96
97
98
99
100
101
102
103
104
105
106
107
108
109
110
111
112
113
114
115
116
117
118
119
120
121
122
123
124
125
126
127
128
129
130
131
132
133
134
135
136
137
138
139
140
141
142
143
144
145
146
147
148
149
150
151
152
153
154
155
156
157
158
159
160
161
162
163
164
165
166
167
168
169
170
171
172
173
174
175
176
177
178
179
180
181
182
183
184
185
186
187
188
189
190
191
192
193
194
195
196
197
198
199
200
201
202
203
204
205
206
207
208
209
210
211
212
213
214
215
216
217
218
219
220
221
222
223
224
225
226
227
228
229
230
231
232
233
234
235
236
237
238
239
240
241
242
243
244
245
246
247
248
249
250
251
252
253
254
255
256
257
258
259
260
261
262
263
264
265
266
267
268
269
270
271
272
273
274
275
276
277
278
279
280
281
282
283
284
285
286
287
288
289
290
291
292
293
294
295
296
297
298
299
300
301
302
303
304
305
306
307
308
309
310
311
312
313
314
315
316
317
318
319
320
321
322
323
324
325
326
327
328
329
330
331
332
333
334
335
336
337
338
339
340
341
342
343
344
345
346
347
\begin{question}
我们班有26名男同学,23名女同学,按一小组最多六人来计算,全班最多可以分为几
个小组?
\end{question}
\begin{solution}
\begin{align*}
26 + 23 = 49\text{(名)}
49 \div 6 = 8 \cdots \cdots 1 \text{(个)}
\end{align*}
最多可以分为9个小组。
\end{solution}
\begin{question}
求不定积分$\displaystyle\int e^{2x}\,(\tan x+1)^2 dx$。
\end{question}
\begin{solution}
\everymath{\displaystyle}%
原式$=\int e^{2x}\,\sec^2 x dx+2\int e^{2x}\,\tan x dx$ \score{2}
\hspace{5em}${}=\int e^{2x}\,d(\tan x)+ 2\int e^{2x}\,\tan x dx$ \score{4}
\hspace{5em}${}= e^{2x}\,\tan x - 2\int e^{2x}\,\tan x dx+ 2\int e^{2x}\,\tan x dx$ \score{6}
\hspace{5em}${}= e^{2x}\,\tan x + C$ \score{8}
\end{solution}
\begin{question}
求过点$A(1,2,-1), B(2,3,0),C(3,3,2)$ 的三角形$\triangle ABC$ 的面积和它们确定的平面方程.
\end{question}
\begin{solution}
由题设$\overrightarrow{AB}=(1,1,1),\overrightarrow{AC}=(2,1,3)$, \score{2}
故$\overrightarrow{AB}\times \overrightarrow{AC}=\begin{vmatrix}
\vec{i}&\vec{j} &\vec{k}\\
1&1&1\\
2&1&3\\
\end{vmatrix}=(2,-1,-1)$, \score{4}
三角形$\triangle ABC$ 的面积为$S_{\triangle ABC}=\dfrac{1}{2}\big|\overrightarrow{AB}\times
\overrightarrow{AC}\big|=\dfrac{1}{2}\sqrt{6}.$ \score{6}
所求平面的方程为$2(x-2)-(y-3)-z=0$, 即$2x-y-z-1=0$ \score{8}
\end{solution}
\begin{question}
计算四阶行列式 $A = \left|\begin{array}{cccc}
0 & 1 & 2 & 3\\
1 & 2 & 3 & 0\\
2 & 3 & 0 & 1\\
3 & 0 & 1 & 2
\end{array}\right|$ 的值.
\end{question}
\begin{solution}
$A = \left|\begin{array}{cccc}
0 & 1 & 2 & 3\\
1 & 2 & 3 & 0\\
2 & 3 & 0 & 1\\
3 & 0 & 1 & 2
\end{array}\right| = \left|\begin{array}{cccc}
0 & 1 & 2 & 3\\
1 & 2 & 3 & 0\\
0 & - 1 & - 6 & 1\\
0 & - 6 & - 8 & 2
\end{array}\right| = 1 \cdot (- 1)^{2 + 1} \left|\begin{array}{ccc}
1 & 2 & 3\\
- 1 & - 6 & 1\\
- 6 & - 8 & 2
\end{array}\right|$ \score{4}
\qquad $= -\left|\begin{array}{ccc}
1 & 2 & 3\\
0 & - 4 & 4\\
0 & 4 & 20
\end{array}\right| = - \left|\begin{array}{cc}
- 4 & 4\\
4 & 20
\end{array}\right| = -(-4\cdot20-4\cdot4) = 96$ \score{8}
\end{solution}
\begin{question}
用配方法将二次型 $f = x_1^2 + 2 x_1 x_2 - 6 x_1 x_3 + 2 x_2^2 - 12
x_2 x_3 + 9 x^2_3$ 化为标准形 $f = d_1 y^2_1 + d_2 y^2_2 + d_3 y^2_3$ .
\end{question}
\begin{solution}
$f = x_1^2 + 2 x_1 x_2 - 6 x_1 x_3 + 2 x_2^2 - 12 x_2 x_3 + 9 x^2_3$ \par
\qquad$= x_1^2 + 2 x_1 (x_2 - 3 x_3) + (x_2 - 3 x_3)^2 + x_2^2 - 6 x_2 x_3 $ \par
\qquad$= (x_1 + x_2 - 3 x_3)^2 + x_2^2 - 6 x_2 x_3$ \score{3}
\qquad$= (x_1 + x_2 - 3 x_3)^2 + x_2^2 - 2 x_2 \cdot 3 x_3 + (3 x_3)^2 - 9x_3^2$ \par
\qquad$= (x_1 + x_2 - 3 x_3)^2 + (x_2 - 3 x_3)^2 - 9 x_3^2$ \score{6}
令$y_1 = x_1 + x_2 - 3 x_3, y_2 = x_2 - 3 x_3, y_3 = x_3$, \newline
则$f = y_1^2 + y_2^2 - 9y_3^2$为标准形.\score{8}
\end{solution}
\begin{question}
设每发炮弹命中飞机的概率是0.2且相互独立,现在发射100发炮弹.\par
用切贝谢夫不等式估计命中数目$\xi$在10发到30发之间的概率.\par
用中心极限定理估计命中数目$\xi$在10发到30发之间的概率.
\end{question}
\begin{solution}
$E\xi = n p = 100 \cdot 0.2 = 20, D\xi = n p q = 100 \cdot 0.2 \cdot 0.8 = 16$. \score{2}
$P (10 < \xi < 30) = P (| \xi - E \xi | < 10) \ge 1 - \frac{D\xi}{10^2}
= 1 - \frac{16}{100} = 0.84$. \score{4}
$P (10 < \xi < 30) \approx \Phi_0 \left( \frac{30 - 20}{\sqrt{16}}\right)
- \Phi_0 \left( \frac{10 - 20}{\sqrt{16}} \right)$ \score{6}
\qquad $= 2 \Phi_0 (2.5) - 1 = 2 \cdot 0.9938 - 1 =0.9876$ \score{8}
\end{solution}
\begin{question}
从正态总体$N(\mu,\sigma^2)$中抽出样本容量为16的样本,算得其平均数为3160,标准差为100.
试检验假设$H_0:\mu=3140$是否成立($\alpha = 0.01$).
\end{question}
\begin{solution}
待检假设 $H_0 : \mu = 3140$. \score{1}
选取统计量 $T = \frac{\bar{X}-\mu}{S / \sqrt{n}} \sim t(n-1)$. \score{3}
查表得到 $t_{\alpha} = t_{\alpha} (n - 1) = t_{0.01} (15) =2.947$. \score{5}
计算统计值 $t = \frac{\bar{x} - \mu_0}{s/\sqrt{n}} =\frac{3160-3140}{100/4} = 0.8$.\score{7}
由于 $| t | < t_{\alpha}$, 故接受 $H_0$, 即假设成立. \score{8}
\end{solution}
\begin{question}
设数列$\{x_n\}$满足$x_1=\sqrt2$,$x_{n+1}=\sqrt{2+x_n}$.证明数列收敛,并求出极限.
\end{question}
\begin{solution}
事实上,由于$x_1<2$,且$x_k<2$时
\[x_{k+1}=\sqrt{2+x_k}<\sqrt{2+2}=2,\]
由数学归纳法知对所有$n$都有$x_n<2$,即数列有上界.
又由于
\[\frac{x_{n+1}}{x_n}=\sqrt{\frac{2}{x_n^2}+\frac{1}{x_n}}>\sqrt{\frac{2}{2^2}+\frac{1}{2}}=1,\]
所以数列单调增加.由极限存在准则II,数列必定收敛.\score{4}
设数列的极限为 $A$ ,对递推公式两边同时取极限得到
\[A=\sqrt{A+2}\]
解得$A=2$,即数列$\{x_n\}$的极限为$2$.\score{8}
\end{solution}
\begin{question}
设事件$A$和$B$相互独立,证明$A$和$\bar{B}$相互独立.
\end{question}
\begin{solution}
$P (A \cdot \bar{B}) = P (A - B) = P (A - A B)$ \score{2}
\qquad $= P (A) - P (A B) = P (A) - P (A) P (B)$ \score{4}
\qquad $= P (A) (1 - P (B)) = P (A) P (\bar{B})$ \score{6}
所以$A$和$\bar{B}$相互独立.\score{8}
\end{solution}
\begin{question}
计算四阶行列式 $A = \left|\begin{array}{cccc}
0 & 1 & 2 & 3\\
1 & 2 & 3 & 0\\
2 & 3 & 0 & 1\\
3 & 0 & 1 & 2
\end{array}\right|$ 的值。
\end{question}
\begin{solution}
$A = \left|\begin{array}{cccc}
0 & 1 & 2 & 3\\
1 & 2 & 3 & 0\\
2 & 3 & 0 & 1\\
3 & 0 & 1 & 2
\end{array}\right| = \left|\begin{array}{cccc}
0 & 1 & 2 & 3\\
1 & 2 & 3 & 0\\
0 & - 1 & - 6 & 1\\
0 & - 6 & - 8 & 2
\end{array}\right| = 1 \cdot (- 1)^{2 + 1} \left|\begin{array}{ccc}
1 & 2 & 3\\
- 1 & - 6 & 1\\
- 6 & - 8 & 2
\end{array}\right|$ \dotfill 4分\par
\qquad\qquad $= -\left|\begin{array}{ccc}
1 & 2 & 3\\
0 & - 4 & 4\\
0 & 4 & 20
\end{array}\right| = - \left|\begin{array}{cc}
- 4 & 4\\
4 & 20
\end{array}\right| = -(-4\cdot20-4\cdot4) = 96$ \dotfill 8分
\end{solution}
\begin{question}
用配方法将二次型 $f = x_1^2 + 2 x_1 x_2 - 6 x_1 x_3 + 2 x_2^2 - 12
x_2 x_3 + 9 x^2_3$ 化为标准形 $f = d_1 y^2_1 + d_2 y^2_2 + d_3 y^2_3$ 。
\end{question}
\begin{solution}
$f = x_1^2 + 2 x_1 x_2 - 6 x_1 x_3 + 2 x_2^2 - 12 x_2 x_3 + 9 x^2_3$ \par
\qquad\qquad$= x_1^2 + 2 x_1 (x_2 - 3 x_3) + (x_2 - 3 x_3)^2 + x_2^2 - 6 x_2 x_3 $ \par
\qquad\qquad$= (x_1 + x_2 - 3 x_3)^2 + x_2^2 - 6 x_2 x_3$ \dotfill 3分 \par
\qquad\qquad$= (x_1 + x_2 - 3 x_3)^2 + x_2^2 - 2 x_2 \cdot 3 x_3 + (3 x_3)^2 - 9x_3^2$ \par
\qquad\qquad$= (x_1 + x_2 - 3 x_3)^2 + (x_2 - 3 x_3)^2 - 9 x_3^2$ \dotfill 6分\par
令$y_1 = x_1 + x_2 - 3 x_3, y_2 = x_2 - 3 x_3, y_3 = x_3$, \newline
则$f = y_1^2 + y_2^2 - 9y_3^2$为标准形。\dotfill 8分
\end{solution}
\begin{question}
设二元随机变量$(\xi, \eta)$的联合分布表为
\begin{tabular}{|l|l|l|l|}
\hline
$\xi \backslash \eta$ & -1 & 0 & 1\\
\hline
0 & 0 & 1/3 & 0\\
\hline
1 & 1/3 & 0 & 1/3\\
\hline
\end{tabular}。\par
(1) 求关于$\xi$和$\eta$的边缘分布。\par
(2) 判断$\xi$和$\eta$的独立性。\par
(3) 判断$\xi$和$\eta$的相关性。
\end{question}
\begin{solution}
(1) 边缘分布为 \begin{tabular}{|l|l|l|}
\hline
$\xi$ & 0 & 1\\
\hline
$P$ & 1/3 & 2/3\\
\hline
\end{tabular}, \ \begin{tabular}{|l|l|l|l|}
\hline
$\eta$ & -1 & 0 & 1\\
\hline
$P$ & 1/3 & 1/3 & 1/3\\
\hline
\end{tabular}. \dotfill 2分 \par
(2) 由$P(\xi = 0, \eta = 0) = \frac{1}{3} \neq \frac{1}{9} = P(\xi = 0) P(\eta = 0)$,
知$\xi$和$\eta$不独立. \dotfill 4分 \par
(3) 由联合分布表求得$\xi \eta$的分布为 \begin{tabular}{|l|l|l|l|}
\hline
$\xi \eta$ & -1 & 0 & 1\\
\hline
$P$ & 1/3 & 1/3 & 1/3\\
\hline
\end{tabular}.\dotfill 6分\par
因此有 $\cov(\xi, \eta) = E(\xi\eta) - E\xi E\eta = 0 -\frac{2}{3} \cdot 0 = 0$,
因此$\xi$和$\eta$不相关. \dotfill 8分
\end{solution}
\begin{question}
设随机变量$\xi \sim N (1, 4)$,求$P (- 1 < \xi < 5)$。
\end{question}
\begin{solution}
$P(-1<\xi<5) = \Phi_0\left(\frac{5-1}{2}\right) - \Phi_0\left(\frac{-1-1}{2}\right)$ \dotfill 2分 \par
\qquad $= \Phi_0 (2) - \Phi_0 (- 1)$ \dotfill 4分 \par
\qquad $= \Phi_0 (2) + \Phi_0 (1) - 1$ \dotfill 6分 \par
\qquad $= 0.9773 + 0.8413 - 1 = 0.8186$ \dotfill 8分
\end{solution}
\begin{question}
设每发炮弹命中飞机的概率是0.2且相互独立,现在发射100发炮弹。\par
(1) 用切贝谢夫不等式估计命中数目$\xi$在10发到30发之间的概率。\par
(2) 用中心极限定理估计命中数目$\xi$在10发到30发之间的概率。
\end{question}
\begin{solution}
$E\xi = n p = 100 \cdot 0.2 = 20, D\xi = n p q = 100 \cdot 0.2 \cdot 0.8 = 16$. \dotfill 2分 \par
(1) $P (10 < \xi < 30) = P (| \xi - E \xi | < 10) \geq 1 - \frac{D\xi}{10^2}
= 1 - \frac{16}{100} = 0.84$. \dotfill 4分 \par
(2) $P (10 < \xi < 30) \approx \Phi_0 \left( \frac{30 - 20}{\sqrt{16}}\right)
- \Phi_0 \left( \frac{10 - 20}{\sqrt{16}} \right)$ \dotfill 6分\par
\qquad $= 2 \Phi_0 (2.5) - 1 = 2 \cdot 0.9938 - 1 =0.9876$ \dotfill 8分
\end{solution}
\begin{question}
从正态总体$N(\mu,\sigma^2)$中抽出样本容量为16的样本,算得其平均数为3160,标准差为100。
试检验假设$H_0:\mu=3140$是否成立($\alpha = 0.01$)。
\end{question}
\begin{solution}
(1) 待检假设 $H_0 : \mu = 3140$. \dotfill 1分\par
(2) 选取统计量 $T = \frac{\bar{X}-\mu}{S / \sqrt{n}} \sim t(n-1)$. \dotfill 3分 \par
(3) 查表得到 $t_{\alpha} = t_{\alpha} (n - 1) = t_{0.01} (15) =2.947$. \dotfill 5分 \par
(4) 计算统计值 $t = \frac{\bar{x} - \mu_0}{s/\sqrt{n}} =\frac{3160-3140}{100/4} = 0.8$.\dotfill 7分 \par
(5) 由于 $| t | < t_{\alpha}$, 故接受 $H_0$, 即假设成立. \dotfill 8分
\end{solution}
\begin{question}
不使用矩阵可相似对角化的判别定理,直接用矩阵的运算和性质证明下面的矩阵$A
=\left(\begin{array}{cc}
1 & 1\\
0 & 1
\end{array}\right)$不能相似对角化,即不存在可逆矩阵$P$和对角阵$\Lambda$使得$P^{-1}AP=\Lambda$。
\end{question}
\begin{solution}
假设有$P = \left(\begin{array}{cc}
a & b\\
c & d
\end{array}\right)$使得$P^{-1}AP = \Lambda$,即$AP=P\Lambda$。\dotfill 2分\par
则有 $$\left(\begin{array}{cc}
a + c & b + d\\
c & d
\end{array}\right) = \left(\begin{array}{cc}
1 & 1\\
0 & 1
\end{array}\right) \left(\begin{array}{cc}
a & b\\
c & d
\end{array}\right) = \left(\begin{array}{cc}
a & b\\
c & d
\end{array}\right) \left(\begin{array}{cc}
\lambda_1 & \\
& \lambda_2
\end{array}\right) = \left(\begin{array}{cc}
a \lambda_1 & b \lambda_2\\
c \lambda_1 & d \lambda_2
\end{array}\right)$$ 因此有 $\left\{ \begin{array}{llll}
a + c & = & a \lambda_1 & (1)\\
b + d & = & b \lambda_2 & (2)\\
c & = & c \lambda_1 & (3)\\
d & = & d \lambda_2 & (4)
\end{array} \right.$ \dotfill 6分\par
由第1个和第3个方程消去$\lambda_1$,可以得到 $c^2 = 0$ 即 $c=0$;
由第2个和第4个方程消去$\lambda_2$,可以得到 $d^2 = 0$ 即 $d=0$。
因此矩阵$P$不可逆,矛盾。\dotfill 10分
\end{solution}
\begin{question}
设事件$A$和$B$相互独立,证明$A$和$\bar{B}$相互独立。
\end{question}
\begin{solution}
$P (A \cdot \bar{B}) = P (A - B) = P (A - A B)$ \dotfill 3分 \par
\qquad $= P (A) - P (A B) = P (A) - P (A) P (B)$ \dotfill 6分 \par
\qquad $= P (A) (1 - P (B)) = P (A) P (\bar{B})$ \dotfill 9分 \par
所以$A$和$\bar{B}$相互独立。\dotfill 10分
\end{solution}